0 Daumen
701 Aufrufe

könnte mir einer hier behiflich sein?

Es sei p > 0. Bestimmen Sie die folgenden Grenzwerte, falls Sie existieren:

limx→∞log(x2+3)/log(x)

! :)

Avatar von

1 Antwort

0 Daumen
 
Beste Antwort

$$ \lim_{x\to\infty}\frac{ln(x^2+3)}{ln(x)}\\=\lim_{x\to\infty}\frac{ln(x^2)+ln(1+3/x^2)}{ln(x)}\\=\lim_{x\to\infty}\frac{2ln(x)+ln(1+3/x^2)}{ln(x)}\\=2+\lim_{x\to\infty}\frac{ln(1+3/x^2)}{ln(x)}\\=2 $$

Avatar von 37 k

Ein anderes Problem?

Stell deine Frage

Willkommen bei der Mathelounge! Stell deine Frage einfach und kostenlos

x
Made by a lovely community